Location via proxy:   [ UP ]  
[Report a bug]   [Manage cookies]                

Math 171, Spring 2010 Please Send Corrections To Henrya@math - Stanford.edu

Download as pdf or txt
Download as pdf or txt
You are on page 1of 4

Homework 4 Solutions

Math 171, Spring 2010


Please send corrections to henrya@math.stanford.edu

P∞ P∞ P∞ p
26.5. Let n=1 an and n=1 bn be absolutely convergent series. Prove that the series n=1 |an bn |
converges.
P∞ P∞ P∞
Solution. Since n=1 |an | and n=1 |bn | converge, by Theorem 23.1, n=1 |an | + |bn | converges.
Note that for all n, we have
p p p
|an bn | = |an ||bn | ≤ (|an | + |bn |)(|an | + |bn |) = |an | + |bn |.
P∞
So by pTheorem 26.3 (Comparison Test), since n=1 |an | + |bn | converges absolutely, we see that
P∞
n=1 |an bn | converges absolutely, and hence converges as all terms are positive.

27.1. Find the radius of convergence R of each of the power series. Discuss the convergence of the power
series at the points |x − t| = R.

Solution.
P∞ x2n−1
(a) n=1 (2n−1)!
Note that
x2(n+1)−1

(2(n+1)−1)!
x2
lim x2n−1 = lim =0

n→∞ n→∞ (2n + 1)(2n)
(2n−1)!

so by the ratio test, the series converges absolutely for all x, and so R = ∞.
P∞ n(x−1)n
(b) n=1 2n
Note that
(n+1)(x−1)(n+1)
n + 1 x − 1 x − 1
2(n+1)
lim = lim =

n(x−1)n n→∞ n 2 2

n→∞
2n

so by the ratio test, the series converges absolutely if |x −P


1| < 2 and diverges if |x − 1| > 2. Thus
∞ P∞
R = 2. If |x − 1| = 2 then the power series diverges, since n=1 n and n=1 (−1)n n diverge.
P∞ n
(c) n=1 n!(x+2)
2n
Note that
(n+1)!(x+2)(n+1) (

2(n+1)
(n + 1)(x + 2) 0 if x = −2
lim = lim =

n→∞ n!(x+2)n n→∞ 2 ∞ otherwise
2n

so by the ratio test, the series diverges if |x + 2| > 0. Thus R = 0. If |x + 2| = 0 then the power
series converges by the ratio test, or by the comment after Definition 27.1.
P∞
(d) n=1 (n1/n − 1)xn
By Theorems 16.7 and 25.1 (Alternating Series Test), the series converges when x = −1.
By equality (16.3) in the proof of Theorem 16.7, we know (1 + n1 )n ≤ n. Taking n-th roots of
P∞
both sides and subtracting 1, we get n1 ≤ n1/n − 1. We know that the series n=1 n1 diverges, hence
P∞
so does n=1 n1/n − 1. Thus the series diverges when x = 1.
1
Hence by Theorem 27.2, we have R = 1, and we have already discussed the behavior of the power
series when |x| = 1.
P∞ n
(e) n=2 (x−π)
n(n−1)
Note that
(x−π)(n+1)
n − 1
(n+1)n
lim (x−π)n = lim (x − π) = |x − π|

n→∞ n→∞ n + 1
n(n−1)
so by the ratio test, the series converges absolutely if |x − π| < 1 and diverges if |x − π| > 1. Thus
P∞ 1
P∞ (−1)n
R = 1. If |x − π| = 1 then the power series converges, since n=1 n(n−1) and n=1 n(n−1) converge.
P∞
27.2. Suppose that the power series
P∞ n=0 an (x − t)n has radius of convergence R. Let p be an integer.
Prove that the power series n=0 n an (x − t)n has radius of convergence R.
p

Solution. We consider thhree cases.


First case: 0 < R < ∞. By Theorem 27.2, we have lim supn→∞ |an |1/n = R1 . Note that
limn→∞ np/n = (limn→∞ n1/n )p = 1p = 1 by Exercise 17.4 and Theorem 16.7. Note also that
{|an |1/n } is a bounded sequence as lim supn→∞ |an |1/n = R1 < ∞. Hence Theorem 20.8 applies, and
we get
1
lim sup(np |an |)1/n = lim sup np/n |an |1/n = lim np/n lim sup |an |1/n =
n→∞ n→∞ n→∞ n→∞ R
P∞ p n
So n=0 n an (x − t) has radius of convergence R.
Second case: R = ∞. The method in the case above works, after replacing R1 with zero.
Third case: R = 0. Hence by Definition 21.2 the sequence {|an |1/n } is not bounded above. Let K
be a positive number. Since limn→∞ np/n = 1, there exists some N such that np/n ≥ 21 for all n ≥ N .
Since {|np an |1/n } is not bounded above, there exists some M > N such that |M p aM |1/M > 2K (for
otherwise max{|1p a1 |1/1 , · · · , |N p aN |1/N , 2K} would be an upper bound for {|np an |1/n }). Hence
|MPp
aM |1/M = M p/M |M p aM |1/M > 12 2K = K. This shows that {|np an |1/n } is not bounded above,

so n=0 np an (x − t)n has radius of convergence 0.

28.1. Prove that the following series converge conditionally.

Solution.
(a) 1 + √12 − √23 + √14 + √15 − √26 + · · ·
Let {an } be the sequence 1, 1, −2, 1, 1, −2, . . . . Let bn = √1n . Note that the sequence of partial
P∞
sums of n=1 an is bounded and {bn } is a decreasing sequence with limit 0. We apply Corollary
28.3 (Dirichlet’s Test) to see that

1 2 1 1 2 X
1 + √ − √ + √ + √ − √ + ··· = an bn
2 3 4 5 6 n=1
converges.
P∞
(b) n=1 (−1)n n(1−n)/n
P∞
Let an = (−1)n n−1 and let bn = n1/n . Note that n=1 an converges by Theorem 25.1 (Alternat-
ing Series Test) and that {bn } is a bounded monotone sequence for n sufficiently large by Theorem
16.7. We apply Corollary 28.6 (Abel’s Test) to see that

X ∞
X ∞
X
(−1)n n(1−n)/n = (−1)n n−1 n1/n = an bn
n=1 n=1 n=1
converges.

2
P∞
28.3. Give anPexample of a convergent series n=1 an and a positive sequence {bn } such that limn→∞ bn =

0 and n=1 an bn diverges.
(
(−1)n 0 n odd
Solution. Let an = √n and bn = .
√1 n even
P∞ n
Then
P∞ n=1 anPconverges by Theorem 25.1 (Alternating Series Test) and limn→∞ bn = 0. Note
∞ 1
n=1 an bn = m=1 2m diverges by Corollary 24.3.
Pp
28.6. Let {an } be a periodic sequence, an = an+p for all n and
P∞for fixed p, for which n=1 an = 0. Let
{bn } be a decreasing sequence with limit 0. Prove that n=1 an bn converges.
P∞ Pp
Solution. Consider the partial
Psums sn of the series n=1 an . Since {an } is periodic and n=1 an =
p
0, we see that sn+p = sn + i=1 an+i = sn for P∞ all n. Hence sn ∈ {s1 , ..., sp } for all n. This shows
that the sequence of partial sums of
P∞ the series n=1 an is bounded. Hence Corollary 28.3 (Dirichlet’s
Test) applies, and we conclude n=1 an bn converges.

xn
P∞
29.3. Prove that n=0 n! converges absolutely for every x and
∞ ∞ ∞
X xn X y n X (x + y)n
( )( )=
n=0
n! n=0 n! n=0
n!

Solution. Note that


xn+1
x
(n+1)!
lim n = lim =0

n→∞ x n→∞ n + 1
n!

so by the ratio test, the series converges absolutely for every x.


n n
By letting an = xn! and bn = yn! in Theorem 29.9, we get that
∞ ∞ ∞ ∞
X xn X y n X X
( )( )=( an )( bn )
n=0
n! n=0 n! n=0 n=0

X
= cn
n=0
X∞ n
X
= ( ai bn−i )
n=0 i=0
∞ X n
X xi y n−i
= ( )
n=0 i=0
i! (n − i)!
∞ n  
X 1 X n i n−i
= ( xy )
n=0
n! i=0 i

X (x + y)n
=
n=0
n!

where the last equality is by the binomial theorem.


P∞ P∞
29.5. Give an example of convergent series n=0 an and n=0 bn whose Cauchy product diverges.

Solution.
3
P∞ P∞ P∞ 1
Let n=0 an = n=0 bn = n=0 (−1)n √n+1 . This series converges by Theorem 25.1 (Alternating
Series Test). The n-th term in the Cauchy product is
n n n
X X 1 1 X 1 1
cn = ak bn−k = (−1)k (−1)n−k √ √ = (−1)n √ √
k=0 k=0
k + 1 n + k + 1 k=0
k + 1 n − k+1
Since √1 ≥ √1 and likewise √ 1
≥ √1 , we have
k+1 n+1 n−k+1 n+1
n
X 1 1
|cn | ≥ = (n + 1) =1
n+1 n+1
k=0
P∞
So limn→∞ cn 6= 0, so the series n=0 cn diverges by Theorem 22.3.

You might also like